Finite Difference, Discrete MacLaurin, dan Integer-Valued Polynomials

[Pelatihan Tim IMO 2019 Indonesia , Soto Bang Sawit]

Sebuah monomial $M_k(z)=z^k$ dapat kita gunakan dalam expansi Maclaurin-Taylor dari fungsi $f$  sebagai  berikut:

\[f(z) = \sum_{k=0}^{\infty} a_k M_k(z) \]

dimana $a_k = \frac{d^k (M_k(z))}{dz}\big\lvert_{z=0}$,  yakni merupakan turunan ke-$k$  dari $M_k(z)$ yang dievaluasi di $z=0$.

Sekarang definisikan barisan suku-banyak sebagai berikut:

\[P_0(z)=1 \qquad P_{1}(z)=z \qquad  P_2(z)=z(z-1) \qquad  P_3(z)=z(z-1)(z-2) \cdots\]

Secara umum definisinya adalah sebagai berikut

\[P_k(z)=z(z-1)\cdots (z-k+1)\]

Untuk "turunan"-nya , kita akan menggunakan finite difference dari fungsi $f$, yakni
\[\Delta f(z) = f(z+1)-f(z) \]

Perhatikan bahwa operator $\Delta (g(z))$ bersifat linear, yakni $\Delta(C  \cdot g(z)) = C \cdot \Delta (g(z))$ dan $\Delta (g_1(z)+g_2(z)) = \Delta (g_1(z)) + \Delta (g_2(z))$.


Dengan melakukan finite difference pada suku-banyak $P_k(z)$, kita  dapat memperoleh:

\[\Delta P_k(z) =  k P_{k-1}(z).\]

Relasi diatas cukup similar dengan rumus turunan $\frac{d}{dz}(M_k(z)) = k M_{k-1}(z)$.

Dengan bekal properti diatas,  kita akan mencoba untuk mendapatkan sebuah  expansi dari suatu fungsi $f$, yang "analog" dengan  expansi Maclaurin-Taylor. (bahkan penurunannya juga  mirip.)

Expansi berikut disebut  Newton Series:

\[f(z)=\sum_{k=0}^{\infty} a_k P_k(z) = a_0 + a_1 z +a_2 (z)(z-1) + a_3 z(z-1)(z-2)+ \cdots\]

Perhatikan bahwa

\begin{align*}\Delta^2 f(z) &= \Delta ( \Delta f(z) ) \\  &= \Delta ( f(z+1)-f(z)) = (f(z+2)-f(z+1)) - (f(z+1)-f(z)) \\ &= f(z+2) -2f(z+1)+f(z) \end{align*}


\begin{align*}\Delta^3 f(z)  &= \Delta (f(z+2) -2f(z+1)-f(z))\\ &= f(z+3)-2f(z+2)+f(z+1) - f(z+2)+2f(z+1)-f(z) \\ &=f(z+3)-2f(z+2)+3f(z+1)-f(z) \end{align*}

Pola koefisien diatas menurut pada koefisien binomial, dan hal ini  cukup mudah untuk  dibuktikan dengan induksi, yakni

\[\Delta^n (f(z)) = \sum_{k=0}^n (-1)^{n-k}{n \choose k}f(z+k) \]

Untuk $z=0$, kita peroleh

\begin{equation}\Delta^n (f(0)) =\sum_{k=0}^n(-1)^{n-k} {n\choose k} f(k)  \qquad   \qquad (1) \end{equation}

Sekarang kembali lagi ke  Newton Series $f(z)=\sum_{k=0}^n a_k P_k(z)$.  Perhatikan bahwa karena $P_k(0)=0$ untuk $k\geq 1$, maka $f(0)=a_0$.  Lalu dengan melakukan finite difference ke fungsi  $f(z)$, diperoleh

\[\Delta (f(z)) = \sum_{k=0}^{\infty}  a_k \Delta P_k(z)  = \sum_{k=0}^n  a_k \cdot k \cdot P_{k-1}(z) = \sum_{k=1}^{\infty} k  a_ k  P_{k-1}(z)\]

Sehingga $\Delta (f(0)) = a_1$. 

Lebih lanjut diperoleh

\[\Delta^2 (f(z)) = \sum_{k=1}^{\infty} ka_k (k-1) P_{k-2}(z) = \sum_{k=2}^{\infty} ka_k (k-1) P_{k-2}(z).\]

Sehingga $\Delta^2 (f(0)) = 2 a_2$. 

Dengan menggunakan induksi dan cara diatas, dapat diperoleh bahwa $\Delta^n(f(0)) = n!  a_n$, sehingga

\[f(z)= \sum_{n=0}^{\infty}  \frac{\Delta^n(f(0))}{n!} P_n(z) \]

jadi
\[f(z)=\sum_{n=0}^{\infty}  \frac{\Delta^n(f(0))}{n!} z(z-1)  \cdots  (z-n+1)    \qquad \qquad (2)\]

voilà!

Untuk kasus khusus  ketika $f(z)=c_m z^m + c_{m-1} z^{m-1} + \cdots +c_0$, yang  merupakan  suku-banyak dengan derajat $m$, maka suku-suku pada expansi  diatas harus  berhenti, yakni pada pangkat tertinggi  dari $f(z)$, sehingga diperoleh

\[f(z)= \sum_{n=0}^m \frac{\Delta^n(f(0))}{n!} z(z-1)  \cdots  (z-k+1)  \]

Dengan menyamakan  koefisien  dan  persamaan (1),  kita peroleh

\[m! c_m = \Delta^m (f(0)) = \sum_{k=0}^m (-1)^{m-k} {m \choose k} f(k) \]

dimana $c_m$ adalah koefisien tertinggi  dari suku-banyak $f(z)$.

Perlu diketahui juga bahwa himpunan suku-banyak $\{P_k(X)\}_{k \geq 0}$ merupakan himpunan yang bebas linear di $\mathbb{Q}$.  Karena jika

\[\alpha_1 P_{k_1}(X)+\cdots + \alpha_m P_{k_m}(X)=0\]

dimana asumsikan $k_1 > k_2 >  \cdots > k_n$ dan $\alpha_i \in \mathbb{Q}$, kita peroleh dengan substitusi $t=k_n$, maka diperoleh $P_{k_{j}}(t) = 0$ untuk $j < n$,  dan $P_{k_n}(k_n)= (k_n)! \neq 0$, sehingga

\[\alpha_n (k_n)!  = 0 \Rightarrow \alpha_n =0\]

apabila dilanjutkan dengan $t=k_{n-1}$, dan seterusnya, maka diperoleh semua $\alpha_i = 0$. Jadi $\{P_k(X)\}_{k \geq 0}$ bebas linear.



Expansi diatas juga dapat memberikan syarat cukup dan perlu untuk sebuat suku banyak $P(X)$ dengan koefisien rational yang memenuhi $P(n) \in \mathbb{Z}$ untuk setiap $n \in \mathbb{Z}$, atau biasa disebut integer-valued polynomials.  Contoh $P(n)=\frac{n(n+1)}{2}$.

Sebuah  suku-banyak $P(X) \in \mathbb{Q}[X]$ dimana $deg(P(x))=n$ merupakan suku-banyak integer-valued  jika dan hanya jika $\Delta^k P(x) \in \mathbb{Z}$ untuk setiap bilangan bulat $k=0,2,..,n$.  Jika dan hanya jika \[P(X) = \sum_{j=0}^n a_j {X \choose j} \quad \qquad \text{dimana }a_j \in \mathbb{Z}\]

Buktinya cukup mudah, Jika $P(x)$ integer valued, maka dari rumus $\Delta^n (P(0)) =\sum_{k=0}^n(-1)^{n-k} {n\choose k} P(k)$, kita peroleh $\Delta^n (P(0))$ selalu bilangan bulat. Sebaliknya jika $\Delta^k P(x) \in \mathbb{Z}$ untuk setiap bilangan bulat $k=0,2,..,n$, maka karena ${X \choose j}$ selalu bilangan bulat untuk setiap $j$, diperoleh  $P(X) = \sum_{j=0}^n a_j {X \choose j} \quad \qquad \text{dimana }a_j \in \mathbb{Z}$, dengan $a_j = \Delta^j (P(0))$.




Berikut ini adalah salah satu penggunaan identitas (2) pada soal USA TST 2019

Misalkan $\mathbb{Z}/n\mathbb{Z}$ menyatakan himpunan bilangan  bulat dalam modulo $n$ (jadi mempunyai $n$  anggota). Tentukan semua bilangan asli $n$ sedemikian sehingga terdapat fungsi bijektif $g: \mathbb{Z}/n \mathbb{z} \rightarrow \mathbb{Z}/n \mathbb{Z}$ sedemikians sehingga 101 buah fungsi
\[g(x) , \quad  g(x)+x , \quad g(x)+2x ,  \quad , \cdots, \quad ,  g(x)+100x\]
juga merupakan fungsi bijektif.

Solusi berikut  merupakan solusi dari Evan Chen.  Tapi soal ini sebenarnya tidak begitu susah apabila kita sudah mengetahui rumus penjumlahan $\sum_{k=1}^m k^j$,  yang melibatkan bilangan  Stirling, dan juga dapat diturunkan dari rumus Newton Series di atas.

Intinya, seperti biasa , dalam mengerjakan soal, kita coba lihat untuk kasus kecil dulu, yakni ketika $l=101$ diganti, misalkan $l=2$ atau $l=3$.

Pada kasus $l=2$ kita ingin fungsi $g(x)$ dan $g(x)+x$ merupakan fungsi bijektif, atau permutasi dari $(1,2,...,n)$. Jadi apabila kita tambahkan diperoleh

\[\sum_{k=1}^n g(k) = \frac{n(n+1)}{2} \qquad \sum_{k=1}^n  g(x)+x = \frac{n(n+1)}{2}\]

sehingga diperoleh $\frac{n(n+1)}{2} + \frac{n(n-1)}{2} \equiv \frac{n(n+1)}{2} \pmod n$.

jadi  $\frac{n(n+1)}{2} \equiv 0 \pmod n$, sehingga $n$ bilangan ganjil.

Untuk tiga buah fungsi  $g(x)$,  $g(x)+x$, dan $g(x)+2x$ kita bisa buat menjadi

\[g(x)^2  - 2(g(x)+x)^2 + (g(x)+2x)^2 = 2x^2\]

Sehingga

\begin{align*}\sum_{k=1}^n 2k^2 &= \sum_{k=1}^n g(k)^2 - 2 \sum_{k=1}^n (g(k)+2k)^2 + \sum_{k=1}^n (g(k)+2k)^2\\ &= \sum_{k=1}^nk^2 - 2  \sum_{k=1}^n k^2  + \sum_{k=1}^n k^2  \\ &=0  \end{align*}

Jadi $\frac{(n)(n+1)(2n+1)}{3} \equiv 0  \pmod n$, sehingga disimpulkan  $3\nmid n$.

Sebenarnya dapat dilihat bahwa pola yang dibutuhkan adalah membentuk $g(x)$, $g(x)+x$, $g(x)+2x$, ...,  $g(x)+kx$ menjadi $k!x^k$.   Dan pola ini berdasarkan dua kasus kecil diatas, seperti baris pada segitiga pascal (ganti tanda) yakni:  $(1,-1)$ dan $(1,-2,1)$.

Untuk membuktikan pola ini, kita dapat menggunakan rumus (2) diatas pada fungsi $w(k)=x\cdot k + g(x)$, yaitu merupakan fungsi linear dalam variabel $k$. Pertatikan  bahwa  $w(k)^m$  merupakan suku-banyak dalam $k$ dan berderajat $m$, dengan koefisien tertinggi sama-dengan  $x^m$. Jadi  apabila kita pakai rumus  diatas diperoleh

\[m! x^m = \Delta^m (w(0)^m) = \sum_{k=0}^m (-1)^{m-k} {m \choose k} w(k)^m  =    \sum_{k=0}^m (-1)^{m-k} {m \choose k}  (g(x)+k\cdot x)^m \]

Jadi

\[m! \sum_{x \in \mathbb{Z}/n\mathbb{Z}} x^m  \equiv \sum_{k=0}^m  (-1)^{m-k} {m \choose k} \left( \sum_{x \in \mathbb{Z}/n\mathbb{Z}}(g(x)+k\cdot x)^m\right)   \]

Sedangkan dari hipotesa  $(g(x)+k\cdot x)^m$ bijektif kita peroleh

\[\sum_{x \in \mathbb{Z}/n\mathbb{Z}}(g(x)+k\cdot x)^m  \equiv  0^m + 1^m + \cdots+ (n-1)^m \equiv l_{n-1}\]

untuk setiap $k$.

Jadi diperoleh

\[m! \sum_{x \in \mathbb{Z}/n\mathbb{Z}} x^m \equiv l_{n-1}\left( \sum_{k=0}^m (-1)^{m-k}{m \choose k}\right) \equiv l_n \cdot 0=  0 \pmod n \]

Relasi diatas berlaku untuk $m=1,2,..., 100$, dapat diperiksa bahwa untuk $k=0$, rumus juga berlaku.

Kita akan buktikan bahwa hal  ini menyebabkan $p \nmid n$ untuk setiap bilangan prima $p \leq 101$.

Jika ada bilangan prima $3 \leq p \leq  101$ (karena $p=2$ sudah diperoleh diatas) sehingga $p  \lvert n$, asumsikan $p$ prima terkecil yang demikian, dan $v_p(n)=a$.  Dengan menggunakan $m=p-1$, diperoleh

\[(p-1)! \sum_{x \in \mathbb{Z}/n\mathbb{Z}} x^{p-1} \equiv 0 \pmod n\]

sehingga dari $(p-1)! \neq  0 \pmod {p^a}$  dan $p^a \lvert n$ diperoleh:

\[p^a \big \lvert  \sum_{x \in \mathbb{Z}/n\mathbb{Z}} x^{p-1} \]

Misalkan $S_p \equiv  \sum_{x=1}^{p^a} x^{p-1} \pmod {p^a} $, maka

\[\sum_{k=1}^{p^a} (p^a+k)^{p-1} \equiv \sum_{k=1}^{p^a} (2p^a+k)^{p-1} \equiv \cdots \equiv \sum_{k=1}^{p^a} ((n/p^a-1) p^a+k)^{p-1} \equiv S_p \pmod {p^a}\]

Jadi

\[0 \equiv \sum_{x \in  \mathbb{Z}/n\mathbb{Z}} x^{p-1} \equiv \frac{n}{p^a} \times S_p \pmod {p^{a}}\]

dan karena $p^a \nmid  \frac{n}{p^a}$, diperoleh

\[p^a \big\lvert \sum_{k=1}^{p^a} k^{p-1}\]

Kita akan buktikan ini tidak mungkin untuk $a \geq 1$, yakni dengan menggunakan   induksi matematika, kita akan buktikan bahwa $v_{p}\left( \sum_{k=1}^{p^a} k^{p-1}\right)=a-1$.

Jika $a=1$, maka diperoleh $k^{p-1} \equiv 1 \pmod  p$, untuk $k \neq p$, sehingga

\[\sum_{k=1}^{p} k^{p-1} \equiv p-1 \not \equiv 0 \pmod p.\]
jadi $v_p\left(\sum_{k=1}^{p} k^{p-1}\right)=0$.

Sekarang misalkan benar untuk suatu $a \geq  1$, kita bisa tulis

\[\sum_{k=1}^{p^a} k^{p-1} = \sum_{(k,p^a)  = 1} k^{p-1} + \sum_{k=1}^{p^{a-1}} (k \cdot p)^{p-1}\]


Sekarang misalkan $g$ adalah primitive  root dari $\mathbb{Z}/p^a\mathbb{Z}$, kita peroleh $\{x \, : (x,p^a)=1\} = \{1,g, g^2, ...,  g^{p^{a-1}(p-1)-1}\}$. Sehingga

\[\sum_{(k,p^a)  = 1} k^{p-1} \equiv  1+g^{p-1} + g^{2(p-1)}  + \cdots +g^{(p^a(p-1)-1)(p-1)} \equiv  \frac{(g^{p-1})^{p^{a-1}(p-1)}-1 }{g^{p-1}-1}  \pmod {p^a} \]

Sedangkan karena $p  \lvert g^{p-1}-1$, dengan Lifting The  Exponent diperoleh

\[v_p((g^{p-1})^{p^{a-1}(p-1)}-1)  = v_p(g^{p-1}-1 ) + v_p(p^{a-1}(p-1)) =v_p(g^{p-1}-1 )  + a-1   \]

Jadi

\[v_p \left(\frac{(g^{p-1})^{p^{a-1}(p-1)}-1 }{g^{p-1}-1}  \right)  =  a-1 < a \]

sehingga
\[\sum_{(k,p^a)  = 1} k^{p-1} =  d_1 \cdot p^{a-1} \]
untuk suatu  bilangan $d_1$ yang relatif prima dengan $p$.

Jadi dengan hipotesa induksi

\[\begin{align*}\sum_{k=1}^{p^a} k^{p-1} &= p^{a-1} d_1 + p^{p-1}\left( \sum_{k=1}^{p^{a-1}}  k^{p-1} \right) \\  &=  p^{a-1} d_1 + p^{p-1} \left(d_2 p^{a-2} \right)  \\&= p^{a-1} (d_1 + p^{p-2} d_2)\end{align*}\]

karena $p \nmid d_1 + p^{p-2} d_2$, induksi selesai.

Dengan demikian dapat disimpulkan bahwa tidak ada bilangan prima $p \leq 101$  yang memenuhi $p \lvert n$.

Jadi diperoleh semua bilangan prima yang $\leq  101$ harus tidak membagi $n$. Untuk  contoh fungsi yang memenuhi ini, cukup ambil $g(x)=x$. Maka $g(x)+kx$ harus bijective untuk $1 \leq k \leq 100$. Karena jika $x$  dan $y$ memenuhi $g(x)+kx \equiv g(y)+ky \pmod n$, maka diperoleh $(x-y)(k+1) \equiv 0 \pmod n$,   yang karena  $(k+1,n)=1$, diperoleh $x \equiv y  \pmod n$. Selesai.


Soal berikut ini (yang juga ada pada pelatihan) dapat dikerjakan dengan menggunakan syarat cukup dan perlu untuk suku-banyak bernilai bilangan bulat.

Tunjukkan  bahwa barisan $(\epsilon_n)_{n \in\mathbb{N}}$  yang bernilai plus-minus satu bersifat periodik dengan periode bilangan $2$ berpangkat, jika dan hanya jika $\epsilon_n = (-1)^{P(n)}$ dimana $P(n)$ adalah suku-banyak dengan koefisien bilangan rasional dan bernilai bilangan bulat untuk setiap bilangan bulat.







Continue Reading